Pre-Exam 2021 - legal part: our answers

This was the first e-EQE with the Pre-Exam being held in 4 parts, and with the questions almost fully being taken from the screen (only the calendars in the legal part and the application and the prior art in the claims analysis parts were printable).

In the legal as well as the claims analysis parts, the order of the four statements of each question was randomized, i.e., it was different for different candidates. 

Also the order of the questions was randomized in the legal parts (e.g., a candidate reported getting what is shown below as Q.2 as the first question and getting what is shown as Q.1 below as the third question), but not in the claims analysis parts. [Updated 2.3.2021, 9:20]

On earlier years, we just provided a list of T and F's for the 4 statements per question, as all candidates had the same order of them in the paper exam. In view of the randomized order, we now also provide the statements. If you check your answers with ours, note that the order of the questions and of the four statements from a single question may be different!!!

Please feel invited to comment!

Note: the claims analysis part is discussed in our other blog post: here.
First impressions and general comments can be posted here.

[Update 4 March 2021 7:15:] We have compiled a version of the paper, in English; it is available here. In our version, the sequence of the questions and statements corresponds to the order as was presented to some candidates; other candidates obtained the questions (in the legal parts) and the statements within a question (in the legal parts as well as in the claims analysis parts) in different order. The sequence/numbering of the questions in our version of the paper corresponds to that used in this blog post.

Our answers to the legal questions are given below.

[Update 2 April 2021 8:45:] 
The Examiner's Report was published yesterday on the EQE website (Compendium); all our answers to Q.1-Q.9 correspond to the answers in the Examiner's Report; Q.10 was completely neutralized (see below).
Note that in the Examiner's Report the order of Q.7 and Q.8 is swapped compared to our version, and that the order of the statements in most of the questions is different from our version.


Q.1

The European patent application EP-A was filed on 07 September 2017. The mention of grant of the patent was published in the European Patent Bulletin on 10 February 2021.

The applicant validly filed a divisional application from EP-A (EP-A-DIV) on 05 February 2021.

·         1. The renewal fee for EP-A in respect of the fourth year was due on 07 September 2020.

o   FALSE – last day of month: 30 September 2020 – Art.86(1), R.51(1)

·         2. The renewal fee for EP-A in respect of the third year could have been validly paid on 03 May 2019

o   TRUE – due on 30 Sep 2019, third yr renewal may be paid 6m in advance, so as of 31/3 – R.51(1), de-ultimo-ad-ultimo J4/91

·         3. The renewal fee for EP-A in respect of the fifth year shall be paid to the EPO

o   FALSE – grant was 10/2/21, fifth year starts Sep 2021, so to national offices – Art.86(2), Art.141(1)

·         4. All renewal fees due in respect of EP-A-DIV can be validly paid without an additional fee on 07 June 2021.

o   TRUE – 5/2/21 + 4m [R.51(3)] -> 5/6/21 (Sat) [R.134(1)] -> 7/7/21 (Mon)

 

Q.2

Anna, a French citizen, validly filed a French patent application FR-1 on 06 February 2020. On 08 February 2021 she filed an international application PCT-1 in respect of the same subject-matter as FR-1 at the EPO. Anna intends for PCT-1 to claim priority from FR-1.

·         1. The filing fee for PCT-1 shall be paid directly to the International Bureau.

o   FALSE – need to pay to rO = EPO (FR nat/res, FR language assumed); R.15.1 PCT

o   *The statement cannot be answered without any doubt as the language in which PCT-1 is field by Anna is not specified. If PTC-1 would not have been filed in EN/FR/DE, then EPO will transmit PCT-1 to the IB and the IB is considered the rO (R.19.4(a)(ii))), because EPO as rO does not accept other languages (R.12.1 PCT; R.157(2) EPC).

·         2. PCT-1 can validly claim priority from FR-1.

o   TRUE - 6/2/20 + 12m -> 6/2/21 (Sat) -> 8/2/21; same applicant, same subject-matter; (independent of language being the same or different)

·         3. PCT-1 cannot validly claim priority from FR-1, if FR-1 has been abandoned before the filing of PCT-1.

o   FALSE – Art.4A(3) Paris Convention (PC equivalent of Art.87(3)): Priority can be claimed from FR-1 as long as a filing date has been accorded, whatever may be the subsequent fate of the application.

·         4. Anna can validly file the priority declaration in May 2021

o   TRUE – R.26bis.1 PCT 6/2/20 + 16m -> 6/6/21 (Sun) -> 7/6/21 (Mon)


Q.3

European patent application EP-P was filed as a first filing on 16 December 2018. EP-P was published on 17 June 2020. Application EP-P-DIV1 was filed with the EPO on 06 December 2019 as a divisional application from EP-P. A second divisional application from EP-P (EP-P-DIV2) was filed with the EPO on 06 November 2020. Today, EP-P is still pending. Some of the claims of EP-P-DIV2 contain subject-matter that was not originally disclosed in EP-P.

·         1. EP-P-DIV1 was not validly filed as a divisional application of EP-P, because EP-P-DIV1 was filed before the publication of EP-P.

o   FALSE – “any pending” - R.36(1)

·         2. EP-P will be regarded as prior art under Art. 54(2) EPC against EP-P-DIV2.

o   FALSE – divisional has same date as parent, and the extending matter has no date at all – Art.76(1)

·         3. For EP-P-DIV2 an additional fee as part of the filing fee for a divisional application of a second generation had to be paid.

o   FALSE – divided from same parent, so also a first generation (not filed from EP-P-DIV1)

o   See also Figure in GL A-IV, 1.4.1.1 showing EP2 and EP3 as first generation divisionals out of EP1

·         4. If EP-P was filed in Portuguese and translated into English, EP-P-DIV1 must be filed in English.

o   FALSE – may be filed in PT with translation into English later – R.36(2)

 

Q.4

Applicant A files today, 01 March 2021, an international application, PCT-A, with the EPO. PCT-A claims priority from the applicant’s European patent application, EP-A, filed on 27 February 2020.

·         1. The publication date of PCT-A can be postponed, if the claim to priority is validly withdrawn before completion of the technical preparations for international publication.

o   TRUE – R.90bis.3(d,e) PCT

·         2. For PCT-A to be considered as comprised in the state of the art under Article 54(3) EPC, it is sufficient that PCT-A is published in English.

o   FALSE – also need to pay filing fee - R.165 EPC 

·         3. If PCT-A contains 20 claims, claims fees shall be paid for five claims within one month of the date of filing.

o   FALSE – no claims fees for PCT (and entry is not filing)

·         4. The period for entering the European phase will expire on 02 October 2023.

o   FALSE - 27/2/20 + 31m (R.159(1) EPC) -> 27/9/22 (which is a Tuesday, but no calendar provided for 2022)

o   You would have wrongly concluded TRUE if you started from 1/3/2021 and would have had a 2023 calendar: 1/3/21 + 31m -> 1/10/23 (Sunday, but no calendar provided for 2023 either) [R.134(1)] -> 2/10/23

o    If you answered FALSE because you concluded on 1/10/23, you had the wrong reason and were lucky that the statement was also FALSE.

Note: no calendars were available for 2022 nor 2023, only for 2020 and 2021

 

Q.5

The mention of the grant of the European patent EP-G was published in the European Patent Bulletin. Competitor B filed a notice of opposition against EP-G on the last day of the opposition period.

One month after expiry of the opposition period and following a request from the proprietor of EP-G to competitor Z to cease alleged infringement, competitor Z instituted proceedings for a ruling that competitor Z is not infringing patent EP-G.

·         1. Provided that the opposition proceedings are still pending, competitor Z can file an admissible notice of intervention within three months of the date on which the proceedings for a ruling that competitor Z is not infringing patent EP-G are instituted.

o   TRUE – Art.105(1)(b), R. 89(1), GL D-VII, 6

·         2. If the notice of opposition has not been signed by the opponent or by his representative before expiry of the opposition period, the opposition division shall reject the opposition as inadmissible.

o   FALSE – R.50(3), GL D-IV, 1.2.1(ii): you first get invited

·         3. Competitor B shall file a statement setting out for the first time the grounds of opposition within two months from the date of filing the notice of opposition.

o   FALSE – within 9m of Art.99 (Rule 76(2)), so on this same last day

·         4. If W files third party observations before expiry of the nine-month opposition period, W will be summoned to any oral proceedings taking place before the opposition division.

o   FALSE – W not a party if W only filed TPOs – Art.115

 

Q.6

A European patent application, EP-X, relating to invention X was filed in January 2020. EP-X includes two alternative embodiments: X1 and X2. Embodiment X1 is not sufficiently disclosed as essential technical information is missing. Embodiment X2 is sufficiently disclosed.

·         1. An objection from the examining division under Article 83 EPC can be overcome by restricting EP-X to embodiment X2.

o   TRUE – why not? Amendment overcomes deficiency

·         2. An objection from the examining division under Article 83 EPC regarding embodiment X1 can be overcome by filing additional technical information.

o   FALSE – application shall be enabled; adding tech info would violate 123(2); only informing examiner does not change the application (would work if it is enabled by the examiner considers not; here the question says it IS not)

·         3. If the missing essential technical information regarding embodiment X1 is present in the abstract, the essential technical information can be incorporated into EP-X as missing parts within one month from the date of filing.

o   FALSE – no such provision; also note that abstract is not part of application as filed for 123(2)

o   Note (added 02.03.2021, 8:00): although missing parts can be added within 2 months from filing under Rule 56(2) (at the cost of a redate) so that "essential technical information can be incorporated into EP-X as missing parts within one month from the date of filing", there is no link/relevance of that to the content of the abstract, so there is also no dependency on whether or not "the missing essential technical information regarding embodiment X1 is present in the abstract". Due to the lack of this link, we consider the statement to be False: a statement composed of multiple parts (an argument and a conclusion) can only be valid if both parts are correct and also the argument is in support of the conclusion.

o   [Update 2 April 2021 8:45:] The Examiner's Report does not address Rule 56, nor the relevance of the 1 month in the statement, nor does it address how to asses a statement with a condition (If ..., then ...; see the comments on the blog as to extensive discussions on how to assess such statements). The Examiner's Report only indicates:

"
FALSE - The abstract is not part of the disclosure (Article 85 EPC) so including the essential technical information from the abstract would offend Article 123(2) EPC".

·         4. If the examining division does not raise any objection under Article 83 EPC, lack of sufficiency of disclosure will not be a valid ground for opposition against the patent granted on the basis of EP-X.

o   FALSE – oppo can be based in any ground incl insufficiency Art.100(b)

 

Q.7

The applicant Mr Y filed a European patent application EP-Y. In the summons to oral proceedings, the examining division objected to claim 1 of EP-Y as not having an inventive step in view of document D1 in combination with the skilled person´s common general knowledge.

At the oral proceedings before the examining division, the professional representative of Mr Y is accompanied by Professor N. The professional representative requests that Professor N is allowed to make oral submissions during the oral proceedings. Professor N is a well-known expert in the technical field of the patent application and has recently been awarded a Nobel prize for her work in that technical field.

Moreover, Professor N is accompanied by five of her students who also wish to attend the oral proceedings. Neither the presence of Professor N nor that of her students had been announced prior to the oral proceedings.

·         1. The students of Professor N may attend the oral proceedings as members of the public.

o   FALSE – exam div oral proceedings are not public - Art.116(3); GL E-III, 8.1

·         2. The EPC does not provide for the hearing of witnesses during examination proceedings.

o   FALSE – Art.117(1) allows it in any proceedings before the EPO

·         3. The examining division may admit auxiliary requests filed and signed by Professor N during the oral proceedings.

o   FALSE – professor N is not allowed to represent – Art.134(1)

·         4. Professor N has the right to make oral submissions during the oral proceedings, since she must be considered by the examining division to represent the “person skilled in the art”.

o   FALSE – he is an accompanying person and should have been announced in advance; whether he can speak is at discretion of the division - GL E-III, 8.5

 

Q.8

European patent EP-C was opposed on the grounds of lack of novelty and lack of an inventive step. The opposition division rejected the opposition. The opponent filed an admissible appeal against the decision of the opposition division. Appeal proceedings concerning European patent EP-C are currently pending.

·         1. A fresh ground of opposition under Article 100(b) EPC can now be introduced into the appeal procedure provided that the fresh ground is raised by third party observations under Article 115 EPC, even if the patent proprietor does not agree to its introduction.

o   FALSE – G 10/91, hn 3

·         2. A fresh ground of opposition under Article 100(b) EPC can now be introduced into the appeal procedure provided that the board of appeal considers the ground of opposition as prima facie relevant, even if the patent proprietor does not agree to its introduction.

o   FALSE – G 10/91, hn 3

·         3. A fresh ground of opposition under Article 100(b) EPC can now be introduced into the appeal procedure since the patent proprietor is party to the appeal proceedings as of right.

o   FALSE – G 10/91, hn 3

·         4. A fresh ground of opposition under Article 100(b) EPC can now be introduced into the appeal procedure provided that the fresh ground is raised in an admissible notice of intervention filed during the pending appeal proceedings, even if the patent proprietor does not agree to its introduction.

o   TRUE – G 1/94 (Note: GL D-VII, 6 hints to it, but is directed to first instance )

 

Q.9

European patent application EP-F has been filed in French and oral proceedings have been scheduled before the examining division. At the beginning of the oral proceedings, the professional representative requests to speak in English without providing for interpretation into French. He also requests that the minutes be written in English and declares that he is aware that, if both his requests are permitted, no interpretation and translation into French will be provided, neither for the oral proceedings nor for the minutes.

·         1. During the oral proceedings, the representative can validly file a new amended set of claims in English.

o   FALSE – R.3(2): amendments (which are to be submitted in writing) must be in language of proceedings, so French

·         2. The examining division may reject the request to speak English in the oral proceedings, even though English is an official language of the EPO.

o   TRUE – R.4(1): if the party/representative provides for interpretation, he has the right to use EN; else, it is at the discretion of the division R.4(4)

·         3. The examining division may agree to the request that the minutes be written in English, except for amendments to EP-F, which will be entered in the minutes in French.

o   TRUE (as far as the statements are spoken in English, and provided that the request to speak English was allowed) – R.4(6)

·         4. If the examining division agrees, the representative may make a part of his oral statements during the oral proceedings in Swedish.

o   TRUE – R.4(4)

Note: you can also use the various paragraphs of GL E-V to answer Q.9


Q.10

Applicant B validly files an international application, PCT-B, in 2019 without claiming priority and elects the European Patent Office as competent International Searching Authority (ISA). The European Patent Office is of the opinion that PCT-B does not comply with the requirements of unity of invention and relates to four inventions: B1, B2, B3 and B4. In response to the invitation from the EPO as ISA to pay additional search fees for B2, B3 and B4, applicant A pays additional search fees for B2 and B3 under protest and the protest fee is duly paid. Four months later, applicant A receives the International Search Report.

Note: the reference to Applicant B and later applicant A seems to be an error in the question: in our understanding, reference to a single applicant was intended in the question itself (only the applicant receives the ISR) and to the same applicant in the second statement (only the applicant of the parent can file a divisional). The question is only consistent if "Applicant B" is replaced by "Applicant A", such that all references are to applicant A. (The reference to PCT-B rather than PCT-A can be explained by the earlier reference to a PCT-A in Q.4). Our answers below are based on this correction.

[Update 2 April 2021 8:45:] The Examiner's Report was published yesterday on the EQE website; Q.10 was completely neutralized. The Examiner's Report provides:

"Question 10 mistakenly refers to applicant B and applicant A while it was intended to refer to only one applicant B different from applicant A of question 4. To avoid any disadvantage due to this mistake, it has been decided that for this question 10 all answers are awarded the full marks of 5 points."

·         1. In order to receive a refund of an additional search fee paid in the European phase, the applicant must pay the additional search fee under protest.

o   [Update 2 April 2021 8:45:] NEUTRALIZED acc. Examiner's Report

o   FALSE – protest procedure only exists in international phase, not in EP regional phase

o   See also GL (2019) A-X, 10.2.2 and C-III, 3.3: examining division will, on request, review the justification for charging the search fee in its invitation under Rule 164(2) - no protest fee needed

·         2. If applicant A files a divisional application, EP-B-DIV, for invention B2 at the EPO, after PCT-B validly entered the European phase, the search fee paid for EP-B-DIV shall be refunded at 100%

o   [Update 2 April 2021 8:45:] NEUTRALIZED acc. Examiner's Report

o   FALSE – GL A-X, 10.2.1 + OJ 2019,A4, item 1.2a) : "full refund" but that is 84% for an international search with WO on a PCT application

o   *TRUE can also be argued if you considered applicant A to not be the same applicant as applicant B who filed PCT-B, and considering that neither the question nor the statement specified that the application has been transferred from applicant B to applicant A. The divisional would thus have been filed in the wrong name. The purported divisional application EP-B-DIV that is not filed in the name of the applicant B of the parent application EP-B will not be processed as a European divisional application (GL A-IV, 1.1.3), so that any fees paid will be refunded (GL A-IV, 1.1)

·         3. If the review body of the ISA finds that the protest was entirely justified, the additional search fees will be refunded but not the protest fee.

o   [Update 2 April 2021 8:45:] NEUTRALIZED acc. Examiner's Report

o   FALSE – protest fee also; R.40.2(c) PCT; GL/PCT-EPO A-III, 9.3 or B-VII, 7.2

·         4. If PCT-B has validly entered the European phase without any amendments to the claims, the EPO will invite the applicant to pay a search fee for invention B4 within a period of two months.

o   [Update 2 April 2021 8:45:] NEUTRALIZED acc. Examiner's Report

o   *TRUE if you assume that no amendments are filed in the R 161/162 period – R.164(2). We expect that this was the intended answer.

o   *But this ignores that the R.164(2) communication does not depend on what is claimed when entering the EP phase (i.e., specified under R.159(1)(b)), but depends on what the situation is at the end of the 6m period to (in this case, mandatory) respond to the R.161(1)/R.162 communication. So that:

o   *FALSE can be argued as well: as there is no indication as to where we are in time (inside or outside the R.161(1)/162 period), it will depend on whether it is no longer to file amended claims or whether it is still possible, and, if amended claims are filed, whether B4 will still be among the claims at the end of the 6m period of R.161(1)/162 and which form the basis for the examination as to whether a R.164(2) must be issued or not

o   *FALSE can be argued also by: as there is a non-unity, the applicant must file a response to the search opinion by commenting and/or amending. As the question does not indicate that such response has been filed, the application will be deemed withdrawn at the end of the R.161(1) period, so that no R.164(2) will be issued (unless remedied with FP etc).

o   Note: the 2 month period runs from the notification of the invitation

*Note: our original answer said True - R.164, but due to some valid arguments raised by some candidates, we added why FALSE can also be considered a correct answer [Updated the parts with the *; 5 March 2021]

Note: due to the randomization of the order of the statements, the order is a bit strange here… The third statement would be expected as the first as that is chronologically the earliest. Also, the first statement would be expected after the last, as those two statements are related - then, the risk that the additional fee in statement 1 would be mistaken as a fee payable in the international phase rather than the EP phase would be much reduced compared to the order presented above. Different candidates, who got the statements presented in different order, may thus have experienced different difficulty levels for this question.

Our Pre-Exam 2021 blog is composed of three separate blogs:
  • first impressions with your comments as to first impressions, the Pre-Exam 2021 as a whole, the e-EQE platform, etc
  • legal part with our answers tio the legal questions
  • claims part with our answers to the claims analysis questions
We look forward to your comments!
Comments are welcome in any official EPO language, not just English. So, comments in German and French are also very welcome!

Please do not post your comments anonymously - it is allowed, but it makes responding more difficult and rather clumsy ("Dear Mr/Mrs/Ms Anonymous of 02-03-2021 22:23"), whereas using your real name or a nickname is more personal, more interesting and makes a more attractive conversation. You do not need to log in or make an account - it is OK to just put your (nick) name at the end of your post.

Thanks!

(c) DeltaPatents

Comments

  1. I am still confused about the switch from applicant B to applicant A in question 10. Is this a trick question or just a mistake? To me this question seems unclear.

    ReplyDelete
    Replies
    1. It is an error on the question and in the statement. It must be the sane applicant or there must have been a transfer.,. If B filed the application, A can never receive the ISR.
      Cleat error. Complete question needs to be ignored (5 marks for all candidates for question 10),

      Delete
    2. If the wrong applicant files a divisional, the divisional will not be treated as such and will be considered not filed and all fees paid will be refunded. Then, 10.2 is True.

      But as A received the ISR, A must be the applicant...

      I think the switch from applicant B to A was an error.

      Delete
    3. Hoe well has this exam paper been checked and tested with a test group? Or is that not done?

      Delete
    4. I completely agree with that question 10 must be ignored (5 marks for all candidates). The question is completely wrong and it was hardly impossible to make a sense of it.

      Delete
    5. @Anonymous 2 March 2021 at 02:01:

      All exams are tested with a guinea pig group during the design phase. That group includes members if other EQE Committees. It was indicated at an earlier Tutor meeting that the guinea pig testing with a large group is done once, when the exam is in principle finalized, to spot possible errors, which are then corrected thereafter – but without another full guinea pig test round with the corrected exam paper.

      My guess would be that in this case the testing was the cause for the mistake. A possible scenario could be that the question was first phrased as:

      “Applicant A validly files an international application, PCT-A, in 2019 without claiming priority and elects the European Patent Office as competent International Searching Authority (ISA). The European Patent Office is of the opinion that PCT-A does not comply with the requirements of unity of invention and relates to four inventions: A1, A2, A3 and A4. In response to the invitation from the EPO as ISA to pay additional search fees for A2, A3 and A4, applicant A pays additional search fees for A2 and A3 under protest and the protest fee is duly paid. Four months later, applicant A receives the International Search Report.”

      And that then, in the guinea pig test, it was noted that Question 4 already introduced a PCT-A. As it is usual practice to have unique names for applications throughout the paper, they then changed it here to PCT-B and they also changed the name of the applicant from A to B, as well as the inventions from An to Bn. But… during that process, only the first occurrence of Applicant A was replaced by B, and not the later ones…
      … and then no one noted that the “corrected” version was incorrect.

      Delete
    6. Thanks for your comment Roel. Do you think that examiners will ignore question 10 (5 marks for all candidates)?

      Delete
    7. I was also confused by the Applicant A/B. The question was definitely not clear. It is not expected that errors appear in the exam. In view of that, I think both answers should be allowable.

      Delete
    8. @Anonymous 2 March 2021 at 09:39

      Only the divisional statement refers to "applicant A", the other statements simply refer to "applicant" without specifying A or B.
      So I donot expect that the whole question will be neutralized as you suggest.
      But possibly the statement about the divisional (10.2 in "our" version) will be (as the full search fee will be refunded it the divisional is filed by the wrong applicant).
      I hope we will know shortly!

      Delete
    9. It is possible that if a candidate had an order start from the divisional question, and then he could understand that the following "applicant" all refers to "applicant A" which would make confusion. Hope EPO consideres this.

      Delete
    10. I agree that the order of the statements may have impact on how you understand and answer the questions. In this question, in the legal parts in general, and even more in the claims analysis parts.

      Strictly speaking, "Pre-examination - new instructions and new marking scheme" indicates in 1-a):
      "Each statement within a question is to be considered independently of the other statements."
      So the order of the statements has no effect on the meaning of the statement, and should not effect how you assess a certain statement.
      So, if you follow the instructions strictly, the term "the applicant A" in one statement does not imply that "the [unspecified] applicant" in another statement is also applicant A (although it will usually do, as the term "the applicant" in the statements does of course relate to "the applicant" in the question before all statements).

      But... not all candidates get the statements in the same order in the e-EQE (in contrast to the earlier paper exams, where everybody got the same paper version so the same order), so candidates may perceive different meanings of the individual statements if the order is different. In particular, if in your first statement, the applicant is named "applicant A" and the question mentions both "applicant A" as well as "applicant B" (as here), you may easily and understandably also understand a reference to "a [unspecified] applicant" in the subsequent statements as "applicant A".

      Delete
    11. NB:
      in Mock 1, the questions and statements came in the same order for all candidates (even though the opening screen indicated that they were randomized).
      In Mock 2 and 3 they came, to my surprise, in a randomized order in legal and claims part (even though the opening screen did no longer indicate that they were randomized).
      In the e-EQE itself, questions were still randomized in the legal part, but no longer in the claims part (such that you would no longer start with a problem-solution question in part 3 if you would happen to get Q.16 first). But the order of the statements was still randomized in legal as well as claims parts. The randomized order of the questions (in the legal parts) and the claims (in all 4 parts) is in my view unlucky as is causes different conditions for different candidates: apart from the sequence of the statements within a question having the effect on the interpretation of the various statements, it also the side-effect that some candidates get the least difficult question or statement first and others get the most difficult question or statement first (e.g., the appeal question -Q.8 in our version- in part 2; or the signature statement in the opposition question -Q5 in our question- in part 1), which may have effect not only on the understanding of a question/statement but also on the stress level and psychological state of mind of a candidate. I hope that a better anti-cheat measure will be implemented in a next Pre-Exam.

      Delete
  2. Question 10 Statement 4:
    English version is ambiguous. It is not possible to know if the the 2 months define when the EPO will be sending the invitation (which would be false) or the time limit given to the applicant to perform the payment.

    ReplyDelete
    Replies
    1. Same problem in the French subject

      Delete
    2. 164(2) uses similar wording:

      inform the applicant that a search will be performed in respect of any such invention for which a search fee is paid within a periods of two months;

      Do you consider that ambiguous too?

      Delete
    3. As R.164(2) uses substantially the same wording ("the Examining Division shall inform the applicant that a search will be performed in respect of any such invention for which a search fee is paid within a period of two months"), I do not think you can consider the statement ambiguous.

      Delete
    4. I answered Q 10.4 FALSE because I did not think that the EPO *will* necessarily invite the applicant to pay a search fee under R. 164(2) as stated in the question. After “PCT-B has validly entered the European Phase without any amendments”, I would expect the EPO to send a communication under Rule 161(1) EPC (not a R. 164(2) communication). After expiry of the 6-month period of Rule 161(1) the EPO will invite the applicant to pay further fees pursuant to R. 164(2), if the ED believes the non-unity objection still applies at that point. The question does not indicate that the applicant has waived their right to the R. 161/162 communication, or that the claims were not amended with the 6-month period according to R. 161.
      Thus, the EPO *may* invite the applicant to pay further fees, but there is not enough information in the question to conclude that they *will*.

      Delete
    5. Hi Danielle, I agree
      See my comment (Roel van Woudenberg 3 March 2021 at 16:20) in response to James 2 March 2021 at 10:56 below.

      By the way, the applicant can in this case not simply succesfully waive the 161/162, as there is non-unity: he needs to respond to the deficiencies in the search opinion by commenting or amending.

      Delete
    6. Thx for the discussion on the effect of possible R.161/162-amendments on "will".

      I added a few comments on the R.161/162 complication to the original post.

      Delete
  3. Question 6 Statement 3: I disagree with the proposed solution.

    Missing parts can be filed within 2 months of filing (which includes "within 1 month"). The application will be re-dated, but the sufficiency issue will be solved. The fact that the same information was previously filed in the abstract is irrelevant. I believe the statement is true.

    ReplyDelete
    Replies
    1. I came to the same conclusion.

      Delete
    2. If the abstract half sentence is irrelevant for the second half sentence on missing parts, it renders the statement False.

      Abby

      Delete
    3. I agree with Abby. Statement only true if both parts true and causal / logical relationship correct. Here not, hence false

      Delete
    4. I agree with Abby and Anonymous 2 March 2021 at 01:55 (NB: please use tour name or a nickname).
      I added a comment to our blog post.

      Delete
    5. I thought the correct answer is "false", because there is no indication that the missing information could be considered a "missing part". I always understood Rule 56 to relate to objectively identifiable errors, like having the drawings missing or maybe have page 8 twice in the application but page 9 missing.
      But just adding subject-matter at will to remedy a 83 problem? Maybe I missing something here.

      Delete
    6. I also came to the same conclusion. If the missing essential technical information is present in the abstract, they can incorporate it as the essential technical information into EP-X as missing parts within one month from the date of filing. Why not? If it is available for them (as it is in this case, in the abstract), they can incorporate it as missing parts within two months. Perhaps I am missing something, but this was my reasoning.

      Delete
    7. The question says "if", not "because". Consequently, as a native English speaker, I would interpret this as "in the situation described by the first part, the second part is true". There does not have to be a causal link. As discussed above, the second part is true (i.e. addition of a missing part) irrespective of whether the information is in the abstract or not, consequently I answered true. If the Examiners agree with Roel and Abby, then I think this could be appealed. That said, if DCF is correct and it cannot be seen as a "missing part", then what I have said above becomes irrelevant.

      Delete
    8. [English Speaker] - I answered this as true also. Similar to James, my thought process would have been different if the question said 'because' or 'only if'. However, it does not say this. If the question had said 'If the missing essential technical information regarding embodiment X1 was written on the back of your hand, the essential technical information can be incorporated into EP-X as missing parts within one month from the date of filing'. Whether or not the information was written on the back of my hand has no bearing on whether I can file missing parts. As others have said above, I could be missing something.

      Delete
    9. Apologies for my poorly thought out 'English Speaker' reference. I mean to say that English is my native language.

      Delete
    10. I also answered true. As Adam notes, it would appear to me that the statement would have been true for any 'if' statement regarding the availability of the missing information. In other words, I interpreted this statement as merely an acknowledgement that the missing technical information was available somewhere (this somewhere just happened to be the abstract in this case). I struggle to see how the answer can be false (unless of course this information cannot be incorporated as a missing part for some reason, rendering the second half of the statement false...).

      Perhaps a clearer way of phrasing the question would have been: "the essential technical information can be incorporated into EP-X as missing parts within one month from the date of filing, only if the missing essential technical information regarding embodiment X1 is present in the abstract". Then I would agree that the answer is false.

      Delete
    11. If the statement is True, then what is the relevance of the sentence about the abstract?

      The sentence "If A, then B" implies that A is a condition for B, isn't it? If not, the statement is False.

      Delete
    12. I would say "If A, then B" means that if A is true, then B must also be true. This does not necessarily mean that the reason for B being true is because of A, simply that in all circumstances when A is true, B is also true.

      On a separate note - Roel: What is your opinion on whether the missing information would count as a "missing part"? There is very little information in the EPC as to how "missing part" should be defined, with only J 27/10 providing any guidance.

      Delete
    13. What if the statement had said 'If the missing essential technical information regarding embodiment X1 is present in the abstract, the essential technical information *cannot* be incorporated into EP-X as missing parts within one month from the date of filing'? I would answer that as false because that said information is in the abstract does not prohibit me from filing the same information as missing parts (unless I am missing something). All I did was change 'can' (positive) to 'cannot' (negative). So, the original statement must be true?

      Delete
    14. I spent far too long looking at conditional statements and logic algebra last night - this one was bugging me. I suspect (although I am no expert) that Q6.3 is an example of (or related to) the post hoc fallacy. Is the statement 'if the rooster crows in Eindhoven, in Eindhoven the sun will rise in the morning' true or false? It is an invalid conditional statement because there is no causal link between the rooster crowing and the sun rising. I think this question should be neutralised. Having said that, I would say yes (true) if a Client asked me, within one month filing a European patent, 'can we incorporate that important information originally filed in the abstract into the description as missing parts, I'm neutral as to whether the application is re-dated'.

      Delete
    15. @James3 March 2021 at 16:57
      > I would say "If A, then B" means that if A is true, then B must also be true. This does not necessarily mean that the reason for B being true is because of A, simply that in all circumstances when A is true, B is also true.

      That does not apply for a statement: a statement may say "If A, then B" that statement can be false. For example: "If I file my EP application 16 months after my earlier application, priority can be valid" - that clerly is FALSE.

      > On a separate note - Roel: What is your opinion on whether the missing information would count as a "missing part"? There is very little information in the EPC as to how "missing part" should be defined, with only J 27/10 providing any guidance.

      A missing part is anything from the description or any drawing that was not included in the original filing. If you use R.56(2), you do not need to proof that you originally intended to file that. Also is not so relevant, as you get a redate anyhow (unless you can use the completely contained in the priority version): the redated EP application with the added "missing" pages has the same date and content as a new filing on that redated date.

      Delete
    16. @Anonymous 4 March 2021 at 10:58
      'can we incorporate that important information originally filed in the abstract into the description as missing parts' is a very different statement as the one in the paper. (But yes, you would say "yes" to your client in that case.)

      Delete
    17. @Roel van Woudenberg3 March 2021 at 16:30
      >If the statement is True, then what is the relevance of the sentence about the abstract?

      >The sentence "If A, then B" implies that A is a condition for B, isn't it? If not, the statement is False.

      I am now asking the same questions about the relevance of the statement "within one month from the date of filing"! Is the question implying that this is also a condition for incorporating the missing parts?

      I think ultimately the statement as a whole is open to interpretation (as evidenced by this discussion!). I accept that there is an argument for the conditional link being implicit (which would indeed render the answer false). However, I would equally argue that the following statement, which recites the exact wording of the question, is also correct:

      "It is true that if the missing essential technical information regarding embodiment X1 is present in the abstract, the essential technical information can be incorporated into EP-X as missing parts within one month from the date of filing"

      I will be interested to see how the EPO mark this one.

      Delete
    18. I think it is also important to acknowledge that the question doesn't say 'then'. It says 'If A, B'. A is not posed as an essential condition for B. I would have a different view if the questions said '..., *then* the essential technical information' or '... the essential technical information can *then* be' Rather, I think the question is asking 'if A is true, can B be true'. Not 'only if A is true, can B be true'. I'm looking forward to the Examiner's Report!

      Delete
    19. Likewise I am interested to see what the conclusion is. Irrespective of what is decided for this question, the Examiners should hopefully be a little more careful with the wording in future papers.

      Adam - I agree with your point, however I would say the question is "if A is true, WOULD B be true", rather than "can B be true", since "can B be true" doesn't require B to be true, just that it might be :)

      Delete
    20. This comment has been removed by the author.

      Delete
    21. James, good point. Agreed.

      Delete
    22. @Guy:
      Do you consider the following statement TRUE:
      "If the sun is shining, the EPO is closed on Saturday 6 March 2021"

      Delete
    23. @Roel, I think there are instances where a conditional clause is always true. '[W]hether or not the if clause is true, the independent [or 'main'] clause expresses a continuously true state'. I have read that this is called a 'relevance conditional' - https://www.quickanddirtytips.com/education/grammar/weird-conditionals-if-clauses-that-are-always-true?page=2. See also https://en.wiktionary.org/wiki/relevance_conditional. I would answer your statement "If the sun is shining, the EPO is closed on Saturday 6 March 2021", as well Q6.3 of the pre-exam, as TRUE.

      Delete
    24. @Adam. https://en.wiktionary.org/wiki/relevance_conditional provides:

      Relevance conditional
      1. A subordinate clause, usually introduced by if, that asserts the relevance of the clause to the main clause of the sentence, but not that it entails the main clause; a construction involving such a clause.

      The way I interprete this definition is that "the relevance of the clause to the main clause" is asseted, is that it DOES check whether the IF-part is of relevance for the MAIN clause, i.e., that the IF-part DOES HAVE a logical/causal effect on the main clause.

      So, because the If-part here (what is in the abstract) has no relevance/zeo significance for the main clause (allowing to file it as missing parts, within a certain time limit), I stick to FALSE.

      We will need to what what the official ans will say.

      And, it would be nice if the Committee would give general guidance as how to deal with advanced logical constructs such as this relevance conditional!

      Delete
    25. @Roel. 'but not that it ENTAILs the main clause'. i.e. (not a necessary accompaniment). So, the statements 'if A, B' and 'if not A, B' can both be true. The 'if' clause is, in my view, simply there to confirm that the essential information is available. My instinct was that: When the 'if' clause is conditionally independent of the 'main' clause (I think we agree on that), and the main clause is always true, the answer has to be answered as true. See this wiki page (which of course cannot be taken as absolute) where it is said 'Hence "if I'm a donkey, then two and two is four" is true when translated as a material implication' (https://en.wikipedia.org/wiki/Relevance_logic). So, I think some people (like me) are translating Q6.3 as a material implication (i.e. True), and others are using relevance logic (i.e. False) even though the words 'then' or 'only' are not used in the statement. In any case we all agree on the law but disagree on how a logic statement should be interpreted. Interestingly, I have posed the Donkey statement to a number of [British] family and friends - they all said True! I wonder if there is some sort of bias in the way the statement is usually interpreted depending on nationality. Should be easy enough for the examination officials to determine. Yes, all we can do is wait and see.

      Delete
    26. Material Conditional (https://en.wikipedia.org/wiki/Material_conditional)

      If P, then Q (i.e. P -> Q)

      P Q -- P -> Q

      T T True
      F T True
      F F True
      T F False

      'Any material conditional with a true consequent is itself true but speakers typically reject sentences such as "If I have a penny in my pocket then Paris is in France". These classic problems have been referred to as the paradoxes of material implication, though they are not bona fide paradoxes'

      Delete
    27. Sorry Anonymous, you did not convince me.

      "Material conditional" is a special type of logic, as it clearly defined in that wiki page.

      The page also says:
      "Due to the paradoxes of material implication and related problems, material implication is not generally considered a viable analysis of conditional sentences in natural language."
      So, material implication -the logic that you showed- is not to be applied in normal live. Similar as 1+1 = 2 in normal life and not 10 as it would be in a binary system (or 01 if the Indians are of the other size).

      As you already indicated, elsewhere on that wiki-page it says:

      Material implication does not closely match the usage of conditional sentences in natural language. For example, even though material conditionals with false antecedents are vacuously true, the natural language statement "If 8 is odd then 3 is prime" is typically judged false. Similarly, any material conditional with a true consequent is itself true but speakers typically reject sentences such as "If I have a penny in my pocket then Paris is in France".

      I.e., "If I have a penny in my pocket then Paris is in France" is FALSE in normal situations.

      Delete
    28. But as I said above in "Roel van Woudenberg 8 March 2021 at 10:28" (slightly rephrased):

      It would be nice if the Committee would give general guidance as how to deal with advanced logical constructs such as this possible relevance conditional or material conditions!

      It is a pity if candidates go wrong due to a not commonly understood logical construct of a statement whereas their understanding of the topic is perfect - or if a statement needs to be neutralized as a consequence of that (...).

      Delete
    29. @Adam: You wrote "The 'if' clause is, in my view, simply there to confirm that the essential information is available."

      You have a point, hat is a valid interpretation which can render the statement TRUE, although the logic of the statement is still very uncommon.

      Also, it feels like, if that was meant, that the reference to the abstract is misleading, or a trap if you wish, and the exam committee does not mislead candidates, they just give information that can be used if you go right and some information that can be used if you go wrong. With that in mind, using the abstract as the source of the availability of the ess tech info, while the abstract can -in its identity as being an abstract- not be used as abstract to serve as missing part, but only the information given it can be used as missing part.

      Maybe we cannot agree on what the answer to the statement must be, but we probably can agree that this statement is a good candidate for neutralization?

      Delete
    30. To the German and French speakers: the pre-exam paper is not yet available on the EQE Compendium pages, and we have no others texts than the English version yet: can you inform us what was the wording of the statement in German and French?

      In English it is:
      "If the missing essential technical information regarding embodiment X1 is present in the abstract, the essential technical information can be incorporated into EP-X as missing parts within one month from the date of filing."

      Delete
    31. Wenn die fehlenden wesentlichen technischen Informationen zur Ausführungsform X1 in der Zusammenfassung enthalten sind, können die wesentlichen technischen Informationen innerhalb eines Monats nach dem Anmeldetag als fehlende Teile in EP-X aufgenommen werden.

      Si les informations techniques essentielles manquantes concernant le mode de réalisation X1 figurent dans l’abrégé, les informations techniques essentielles peuvent être incorporées à EP-X en tant que parties manquantes dans un délai d’un mois à compter de la date de dépôt.

      Delete
    32. Not any different to the English version as far I can see.

      I agree with Roel that both interpretations can be reasonable, so neutralization makes sense (and avoiding such wording in future exams).

      Putting aside the formal "IF" logic and wiktionary to one side for a second, my motivation for interpreting the sentence in the way that I, Adam and Guy did was in the context of being a patent attorney (which is what the exam is testing after all). If a client asked me a question of "If the missing information is present in the abstract, then am I allowed to file it as a missing part?", my answer would be "yes, but you would be able to even if it wasn't in the abstract". An answer of "no" would be wrongfully informing the client. I know that this question isn't precisely what was asked in the exam, but this is why I (and presumably others) interpreted the question the way that we did.

      Delete
    33. the website isn't letting me add my name to that last post. Maybe I have commented too often :)

      James

      Delete
    34. @Unknown James: blogger is sometimes unpredictable as to how and whether you can post... if you post with a google account or with a name, the problems are least frequent; if you post anonymously, more frequent. But we have not been ale to identify the exact conditions/causes ;( So if it fails, try again using another option - and check your cookie settings (see "Problems commeting?" in the right menu)

      Delete
    35. James8 March 2021 at 18:29 and Unknown 8 March 2021 at 18:38:
      I am not a native speaker in either of the three languages, but I also consider "Wenn" and "Si" to have the same meaning -and issues- as "If".

      @Unknown 8 March 2021 at 18:38:
      I would also, without doubt, give more or less that same answer: "yes, but not because it is in the abstract, but because we are within 2m from the filing of the application. So we can file a so-called missing part of the description with the information taken from your abstract in it".

      Shall we agree that this statement is a good candidate for neutralization?
      And hope that the competent authority will agree too? ;)

      Delete
  4. Thanks a lot for sharing this so fast. :)

    ReplyDelete
    Replies
    1. And thanks to those candidates that provides us with a copy of their answers so that we could have the paper available quite shortly after the exam.

      Note: if you want a copy of your answers, go to the Wiseflow flows (one-by-one), and at the bottom left you see "Paper" and a small eye next to it. You can print your answers from there. Note that you may need to adjust the scale factor to see the full width of the text.

      Delete
    2. Q5.1 i think it should be false because that is not the first interaction between G and Z. It should be within 3 months from the request to stop infringement.I found it in Visser, so I chose False.

      Delete
    3. The time limit for filing a notice of intervention is given by Rule 89(1) and Article 105(1). Art 105(1)(b) relates to the case in this question - "following a request of the proprietor of the patent to cease alleged infringement, the third party has instituted proceedings for a ruling that he is not infringing the patent.". Rule 89(1) specifies that the notice of intervention must be filed within 3m from when the proceedings (in this case, as specified in Art 105(1)(b)) are instituted - not within 3m from the request to stop infringement.

      Delete
    4. @Vincent: It looks like you interpret a paragraph in reference book wrongly, or did not read the complete relevant paragraph, or there was an error in your reference book, and that you did not check the true legal provision (Art.105).
      Clearly, as Stefanie pointed out, the first interaction between G and Z does not set the time limit: it is when proceedings are instituted, either by the the proprietor (infringement proceedings) or by the alleged infringer (proceedings for a ruling of non-infringement, but only it that is following a request to cease infringement).

      Delete
  5. I disagree with the answer Q1.1 but maybe the French subject is more ambiguous than the English one. Of course the last day for paying the renewal fee is the last day of the month, i.e. 30 september 2020. However, the renewal fee in respect to the fourth year was due from the 30th June to the 30 September. The renewal fee was therefore dur for every day between the 30th June to the 30th Spetember. The 30th Spetember is the deadline ("échéance" in French)

    ReplyDelete
    Replies
    1. Rule 51(1) specifically indicates that the renewal fee for the fourth year cannot be validly paid more than 3 months *before* it is due. During that 3 month period, the renewal fee is not technically "due" yet, but it can be validly paid. This is also clear in the French rule - "Toutes les autres taxes annuelles ne peuvent pas être valablement acquittées plus de trois mois *avant* leur échéance".

      Delete
    2. @Anonymous 2 March 2021 at 09:27:
      to add to the above: you seem to misinterpret the meaning of the word "due" within the context of renewal fees that is given in Rule 51(1):

      "A renewal fee for the European patent application in respect of the coming year shall be DUE ON THE LAST DAY of the month containing the anniversary of the date of filing of the European patent application."
      So term "a renewal fee is due" refers to a single DAY, and not a period of many days.

      Delete
  6. I arrived at different answers for question 8:

    Statement 4 refers to introducing a fresh ground into the appeal procedure without the agreement of the proprietor. It is correct that according to G1/94 the assumed infringer may raise new ground of opposition; however, " if a fresh ground for opposition is raised by the intervener, the case should be remitted to the first instance for further prosecution, unless special reasons present themselves for doing otherwise, for example when the Patentee himself does not wish the case to be remitted" (G1/94; item 13 of the reasons). Thus, according to my understanding, the fresh grounds cannot be introduced into the appeal procedure.

    I also answered statement 3 differently, since it is correct that the patent proprietor is party to the appeal proceedings as of right and this allows him to agree to a new grounds of opposition being introduced into the proceedings. Also, this statement was the only one that didn't include the qualifier "even if the patent proprietor does not agree to its introduction". However, I found this statement very unclear and could have answered either way.

    ReplyDelete
    Replies
    1. I came to the same conclusion for statement 3. It is true that the patent proprietor is party to the appeal proceedings and that fresh ground can be introduced (both statements are true). The patent proprietor being party to the appeal proceedings, they can agree to the introduction of the new ground. I also thought, however, that the statement was very unclear.

      Delete
    2. Statement 8.3 is so unclear. It doesn't actually say who is actually wanting to introduce the fresh grounds. Is it the proprietor or the opponent? The statement simply says that fresh grounds can be introduced since proprietor is party to the proceedings. But it isn't clear to me from this statement that the proprietor wants to introduce the fresh grounds? If proprietor does then I assume the answer is true.

      Delete
    3. I would say that for the question to be answered true, it would need to be true all of the time, consequently it must be false. i.e. simply because the fact that patent proprietor is party to the proceedings doesn't automatically mean that all new grounds of opposition are allowable, which is what the question is asking

      Delete
    4. I agree with James.
      The core of question 8.3 is that it asks whether fresh grounds can be introduced BECAUSE the proprietor is a party as of right.
      He is of course, but that in itself is not a reason to allow fresh grounds.
      A reason is if the applicants allows fresh grounds to be admitted (and if they are found to be highly relevant by the Board).

      Delete
    5. For an "R because C", C must be the cause for the result R.
      So: R, C and the causal link must all be true for the statement to be true.

      G 10/91 does not limit the "power" of the proprietor to not agree to admit fresh grounds to him being a full party , it also applies when he is a party as of right (i..e, when the opposition was rejected by the oppo div, or when the proprietor did not appeal against a decision to main in amended form)

      Delete
  7. Q5.1 I came to False, because according to Visser A105: "an alleged infringer MUST file the notice of intervention within three months (R.89(1)) from the institution of the FIRST ACTION between him and the proprietor"
    If I understood correctly, the first action in the question should be: "a request from the proprietor of EP-G to competitor Z to cease alleged infringement".
    Therefore, the notice of opposition should be filed within 3 months from the date of the request to cease alleged infringement.

    ReplyDelete
    Replies
    1. You should have read the following sentence in the Visser as well, which specifies that said first action is either a procedure for infringement or proceedings for a ruling of non-infringement.

      Delete
    2. Thank you. My bad. I was so nervous and suspicious during the exam. If it were normal practice I would not have stick to the later part after within 3 months :(

      Delete
    3. For this question I refered to T0296/93. "The application starting point for calculating the 3 Month périod for intervention under A105(1) EPC is always the date of the institution of the 1st court action. Where a court action for infringement was first brought by a patentee against an alleged infringer, A105(1) EPC first sentence applies, even though the latter later instituted a court action seeking a declaration of non infringement under A105(1) EPC second sentence, with regard to the same patent"
      So, I answered False...
      Have I missed something ?

      Delete
    4. What was the start of the first court action in your view?

      Reminder: Please use your name, or a nickname.

      Delete
    5. Thanks for your quick reply. I considered it starts from: " request from the proprietor of EP-G to competitor Z to cease alleged infringement"

      I had answered "true" at first but I changed my mind in view of this decision T. :(

      Delete
    6. Hi Elodie,

      Is a letter, or a phonecall, from me to you that says the following already a court action?:

      "Please stop selling your product because I believe that you are infringing on my patent EP-G?"

      Delete
    7. Often made error in exams, where it is regularly tested:

      A warning letter is just a letter and not "an institution of proceedings for infringement".

      There is also case law on this: T 195/93.

      Delete
  8. I also have an issue with the answer for 10.4:

    It clearly states in the Euro-PCT Guide that an invitation to pay additional search fees for non-searched inventions will be issued at the end of the R 161/162 period. The non-searched matter could be removed by the applicant during this period, thus the deciding factor is not whether the claims were amended upon entry into the EP phase under Rule 159(1)(b), but whether the claims have been amended by the end of the R 161/162 period. There is therefore no causal link between the two parts of the question, and I answered false.

    ReplyDelete
    Replies
    1. Hi James,

      you are correct that the applicant has two opportunities to amend at the initial phase of the EP regional phase:
      1) directly at entry under Rule 159(1)(b) ("specify the application documents, as ... or as amended...,...");
      2) before the end of the 6m period given by the notification of the R.161/162 communication.

      So if the applicant enters without any amendments and does not do any amendments (or if he does any amendments, he does not delete B4), he will get a Rule 164(2)(a) invitation in view of the presence of yet unsearched B4.
      But if the applicant enters without any amendments at the 31m date, but subsequently deletes B4 by amendment before the end of the 161/162, he will not get a Rule 164(2)(a) invitation.
      When taking it strictly, the statement thus misses what happens in the 161/162 period, and you cannot answer the statement unambiguously with T or F.
      If you need to choose in the exam between T and F, I think that T is the most natural choice, as there is no indication that the applicant wishes to delete B4.

      Whether this is sufficiently unclear/ambiguous or incorrect for the statement to be neutralized is difficult to say. That you raised the point here may have been of help to get the point to the attention of the Pre-Exam Committee and the Examination Board.

      Delete
    2. There is a clear reason that the Article uses the word "shall" instead of "will". It is true that the info about whether the applicant removes the invention is not given in the question, but, in my view, it leaves both possibilities.
      The EQE is designed to test whether the potential candidates have the ablity to do analysis with mutliple scenirios. If a client comes to an attorney and ask about the above question, and the attorney answered that the EPO "will" give an invitation and not give the possibility that he could choose to accelerate the process by removing the unsearched subject, the attorney is surly not doing a good job.
      There was also "must" questions in the past by asking if some process can be waived. The candidate may understand that this is a similar question to judge if the candidate think that sth will *definitely* happen.
      I hope that examining committee will neutralize this question.

      Delete
    3. Thx for the discussion, you have a very valid point in my view: the effect of the R.161/162 communication may change the "will" to "will not" and the causal relationship is not between "claims at entry" and R.164(2), but between "claims at the end of the 6m period of R.161/162" and R/164(2).
      The answer is thus neither TRUE nor FALSE but "it depends on whether amendments are duly filed in the R161/162 period and, if so, B4 is still among the claims at the end of that period".

      I added a few comments on the R.161/162 complication to the original post.

      Delete
  9. I think the answer to Q5.1 is FALSE for a different reason:

    The statement says ‘provided that the opposition proceedings are still pending’ - this renders the statement FALSE as a notice of intervention can be filed when opposition proceedings are not pending i.e. when appeal proceedings are pending. Any similar thoughts?

    ReplyDelete
    Replies
    1. NO. Although I marked it also FLASE, I don't agree with your argument. “Provided that” should not be undetstood as a necesseity. Instead, you are provided this condition, and have to accept this condition apply to the statement in the following.

      Maybe we are both too paranoid about the language during the exam :(

      Delete
    2. When the appeal is pending, the OPPO is also still considered to be pending. Not sure exactly what the legal basis there is, but Im pretty sure that is how it usually is.
      I know its easy to overthink these kind of things but Patrick is right, the question tells you explicitly to consider it a given that the OPPO is indeed still pending and to base your answer on that.

      Delete
    3. Thanks @DCF, your point reminds me of the retroactive effect about appealing. I think you are right!
      [Not a valid legal opinion] ;)

      Delete
    4. Art.105(1) gives the requirement: Any third party may, in accordance with the Implementing Regulations, **intervene in opposition proceedings** after the opposition period has expired, if the third party proves that [...]

      @M: if you considered appeal, then suspensive effect of the appeal could have led you to conclude that opposition is still pending during opposition appeal. Also t, there is case law on intervention during opposition appeal - that can only be if the opposition appeal is considered pending opposition proceedings as that is the only possibility for intervention ac Art.105.

      Delete
  10. Q3.4: as indicated in GL A‑VII, 1.3, a divisional application must be filed in the language of the proceedings of the parent application.
    Although it can be filed in Portuguese and translated to English, the question recites the wording of the guidelines that it must be filed in English.

    I struggled to chose between T or F.

    ReplyDelete
    Replies
    1. I completely agree. I consider Question 3.4 also ambiguous and I could not choose between TRUE or FALSE.

      If the wording was that the application must ONLY be filed in English, that would have been FALSE.
      I do not see why the question as stated is FALSE.

      Delete
    2. I am completely agree with Marc and Sofia. The answer cannot never be FALSE, as the application MUST be filed in the language of proceedings of the parent. No matter when the application in English is filed, wheater as first filing or by a translation.

      Delete
    3. My reading of the Guidelines you cited (A-VII, 1.3) is that this question actually falls under the SECOND sentence (not the first), which reads:

      "Alternatively, if the earlier (parent) application was not in an official language of the EPO, the divisional application may be filed in the language of the earlier (parent) application"

      i.e. since the parent application was filed in Portuguese (not an official language of the EPO), the Divisional application may be filed in the language of the parent application (i.e Portuguese). Therefore, the Divisional does not NEED to be filed in English in the situation in Q3.4

      Delete
    4. This comment has been removed by the author.

      Delete
    5. I agree with Jay J, as understanding a piece of information alone without the whole context would very likely to lead a wrong choise. As there are so many exceptions and "however", "alternatively" in the EPC realm. Most of the time, you just have to finish reading the whole topic to be sure :(

      Delete
    6. I also thought that Q3.4 was not clearly formulated. The application must be filed in English or may be filed in Portuguese providing a translation in English. For that reason, I think both answers should be accepted.

      Delete
    7. @Peny, your statement contradicts, you literally say: you MUST do A, OR you MAY do B. (if there is an alternative choice, then A is no longer a MUST)

      If you want to CLEARLY FORMULATE your understanding, as you requested the same for the question Q3.4, you should say: you MAY do A, or you MAY do B.

      Then asking yourself: MUST you do A? The answer is clearly NO.

      Delete
    8. Jay J and PPPatrick are correct. The wording in the guidelines is admittedly a little bit unhelpful (use of the word must, followed by an alternative); however, if you read the whole of the paragraph in the guidelines then there is only one interpretation that makes any sense, i.e. filing in English or filing in Portuguese and then filing a translation in English.

      On Daniel's point, if you file the application in Portuguese and file a translation in English, then the application has been filed in Portuguese. The fact that a translation has been filed in English doesn't change this first fact ("filing of translation" does not equal "filing of application")

      Delete
    9. As EP-P is filed in Portugese, the divisional can be :

      - filed in Portugese and subsequently translated into the language of proceedings of the parent, EP-P, so English (witbin 2m, with possibly an additional 2m after invitation under R.58)
      OR:
      - immediately in the language of proceedings of the parent, EP-P, so English

      So, it does not need to be FILED in English and the answer is FALSE.

      Note that the filing of the translation into English after filing the divisional in Portugese is not the filing of the divisional, but the filing of a translation of it! (See Also Art.70(2))

      Delete
    10. @Jay, of course you are right in what you indicate. But anyway, the sentence in GL "European divisional applications must be filed in the language of the proceedings of the earlier (parent) application" makes the question Q3.4 NEVER to be FALSE, even if you file the application in English by means of a translation.

      Delete
    11. Hi, @Roel!. So, according to your interpretation, what is the sense of the sentence in the GL "European divisional applications MUST BE FILED in the language of the proceedings of the earlier (parent) application", IDENTICAL to the stated in the question?. Is so the word "must" badly used in the GL?. Is then the "filed" application indicated of the GL not referred to the filing of the divisional and nevertheless it is in the question?

      Delete
    12. @Daniel Dios, the GL is indeed drafted in a misleading way, with MUST in the first sentence and "Alternatively" in the second sentence. And in the exam, I know we get nervous and sometimes will not check the whole text. However, it is quite obvious that the statement in Q3.4 alone is at least INCOMPLETE, and therefore, false. Lastly, guideline is not a valid legal basis.

      Delete
    13. @Daniel: please also read the next sentence in GL A-IV, 1.3.3.

      Delete
    14. @Daniel

      I think the distinction needs to be made between filing an application in a language and translating the application into a language. Filing in Portuguese and translating to English =/= filing in English.

      I think the GL muddy the waters somewhat by using "must" and then following it by a contradictory option but as a native English speaker, the only logical reading is "you must file in the language of proceedings of the earlier application UNLESS it was not filed in FR/EN/DE, in which case, you can instead file in the language that the earlier application was filed in".

      Therefore, as Roel said above, there are two options: file in Portuguese (and later translate to English) or file in English. Therefore, Q3.4's statement "EP-P-DIV1 must be filed in English" is FALSE - since you have two options, it is *not* true that the Divisional *must* be filed in English.

      Delete
    15. I think there are some similarities with question 4.1 of the pre-Exam 2019. In 2019, the question was if a translation of the application must be filed within 2 months after filing. The examiner's report indicated that the answer must be true since that is literally reflected in R6(1), even though a translation may still be filed within 2m from a R58 invitation. Thus, considering the Examiner's report of pre-Exam 2019, one could assume that also in this question the answer should be true as the question reflects the wording used in the guidelines. Btw the question was successfully appealed.

      Delete
    16. Hi Charlotte, thx for your comment. But you seem to have overlooked that there is a very important difference between 4.1 of pre-exam 2019 and the current case:

      In 4.1 of 2019, non-filing of the translation of the application would result in a R.58 communication inviting you to file the translation within 2 m from its notification - so, you would have a (free) remedy. That caused the discussion to whether one "must" file the translation within the original time limit (2m from filing, Rule 6(1)) -- see D 3/19.

      In the current case, one MUST/SHALL/HAS TO file the divisional immediately in a language allowed under R.36(2). There is no remedy on this case. So, the statement is perfectly fine with the word "must".

      Delete

  11. I think Q9.4 should also be FALSE:

    R.4(4) states: if the parties and the EPO agree, any language may be used.

    I think the answer is therefore FALSE as the statement does not mention that the parties also agree to any language being used. Any similar thoughts?

    ReplyDelete
    Replies
    1. The question relates to oral proceedings before the examining division. These oral proceedings are solely between the applicant party and the EPO - there are no other parties that could not agree to the language. As long as the EPO agrees, the applicant's representative can make statements in Swedish.

      Delete
    2. @stefanie but doesn't the question say 'a part' of the proceedings can be made in swedish? Are you allowed only a part of the proceedings in one language and the other part in another language? I thought you would need to stick to one language you have chosen and for this reason said false to this

      Delete
    3. @Ash - There is no provision that the entirety of the proceedings must be in one language.

      Actually, the wording of Rule 4(6), which relates to the language of the minutes, suggests that multiple different languages may be used.

      Delete
    4. @Stefanie, first of all, I agree with your view on this question, especially it is the language of the minutes, so multiple language may be used.

      However, this situation reminds me of one question in Delta's L-Book, and had marked a T decision in my book:

      T 774/05 reason 6.5 states that "In the view of the Board a party must be clear as to which official language it wishes to use. The party then has a right to both speak and hear in that language, so long as the conditions of Rule 2(1) EPC have been fulfilled. The party does not, however, have a right to have a language in which it will speak and a different language in which it will hear."

      So does this T decision mean that a party has to stick to the languge that has been chosen in the beginning and cannot speak/listen using another language in the oral proceedings?

      Delete
    5. @Patrick: Without knowing the decision in detail, from what you´ve written the situation in the decision appears to differ in several key points, e.g. that it is about EPO official languages and about hearing vs. speaking.

      Most importantly, it says that the party does not HAVE A RIGHT to choose all kinds of different languages. However, the question ask what happens if all parties agree. And in that case, then basically anything goes.

      Delete
    6. Rule 4(4): "If the parties and the European Patent Office agree, any language may be used."

      Also GL E-V, 2:
      "2. Language of a contracting state or other language
      Any party may likewise use one of the official languages of the contracting states, other than English, French or German, on condition that they make provision for interpreting into the language of the proceedings. However, if the parties and the EPO agree, any language may be used in oral proceedings without interpreting or prior notice."

      and GL E-V, 1:
      "The language of the proceedings as defined in Art. 14(3) cannot be changed. This means that any amendments to the application or patent have to be filed in the language of the proceedings (Rule 3(2)).

      If all parties have indicated that they will use another official language, the division may depart from the language of the proceedings so as to manage without or with fewer interpreters (this question normally arises only in opposition proceedings). The parties' summonses are therefore accompanied by information which encourages them to agree how this can be achieved."

      Delete
    7. Thanks for your input Roel. The confusing bit here is "part of his statement", I don't think it was put there without purpose.
      There is no provision allowing a party to oral proceedings to make his statement in multiple languages (different segments of the statement in different languages). All the provisions always refer to changes from one language to another.
      Is this really something that could be allowed?

      Delete
    8. Rule 4 does not indicate that only one language may be used by a party. The language of proceedings may always be used; the R.4(1) language, if conditions satisfied, and the R.4(4) language, if all agreed, may be allowed - in addition, as it does not impose a limitation. This also happens in real life: in The Hague, we sometimes do OP partially in NL and partially in EN (the language of proceedings).

      I can only find one additional restriction in GL E-V:
      GL E-V,1 imposes the restriction, derived from case law, that:
      "A party must be clear as to which official language it wishes to use. It then
      has a right to both speak and hear that language, as long as the conditions
      of Rule 4 have been fulfilled. The party does not, however, have a right to
      have one language in which it will speak and a different language in which
      it will hear (see T 774/05)."

      By the way, I also do not see that limitation that R.4(4) would need to limit to one special language - in my view, it could also be that all parties and the EPO agree on multiple languages. But that is not explicitly documented in the GL (I did not check the case Law Book).

      Delete
  12. First of all, thank you for the answers.

    Regarding Q10.1 I struggle with your answer. As the question, in my opinion, does not ask for a protest fee to be paid. But if the further search fee is paid under protest "muss der Anmelder die zusätzliche Recherchengebühr unter Widerspruch entrichten", so it could also be a communication with the payment that includes a written protest.
    The EPC and guidlines show that a request of the applicant has to be filed. But if the applicant writes "I pay the fees under protest, so pay me my the fees back" isn't that a kind of request and also a kind protest of the applicant?

    ReplyDelete
    Replies
    1. The question asks whether the applicant MUST pay under protest. And the answer to that is clearly no. There is no concept of "protest" in the EPC so he cannot be obliged to protest.
      Your quote from the GL "muss der Anmelder die zusätzliche Recherchengebühr unter Widerspruch entrichten", is related only to the PCT procedure. In the EP phase, there is no such thing as protest.

      Delete
    2. Thank you very much! Now I got it.

      Delete
    3. Also see Guidelines B-VII, 2.1:

      "At the examination stage the applicant may contest the allegation of non-unity and request a refund of one or more of the further fee(s) paid. If the examining division finds this to be justified, the fee(s) in question will be refunded (see, however, B‑XI, 1.2)."

      and GL C-III, 3.1.1 and 3.1.2:

      "3.1.1 No additional search fees paid
      [...]
      It must be taken into account that the final responsibility for establishing whether the application meets the requirement of unity of invention ultimately rests with the examining division (see T 631/97). When considering the issue of unity, the examining division will consider both the reasons given in the search opinion and the applicant's response thereto (see B‑XI, 8, for details of when a response to the search opinion is required); for Euro-PCT cases where no supplementary European search report is prepared, the examining division will consider the reasons given in the WO‑ISA, IPER or supplementary international search report prepared by the EPO and the applicant's response thereto as required by Rule 161(1) (see E‑IX, 3.2). In the absence of any convincing response from the applicant to the issue of unity as raised earlier, the examining division will normally initially uphold the position taken earlier (see B‑XI, 1.2) and will then require deletion of all the inventions other than that which has been searched. If the examining division is convinced, e.g. by arguments from the applicant, that the opinion on unity at the search stage was incorrect, then an additional search is performed for that part of the subject-matter which is judged to be unitary with an invention which was searched (see B‑II, 4.2(iii), and C‑IV, 7.2) and the examination is carried out on those claims which comply with the requirement of unity of invention. The applicant may file a divisional application for any excised subject-matter (see C‑III, 3.2)."

      "3.1.2 Additional search fees paid
      If applicants have taken the opportunity to have other inventions searched, then they may determine that the application is to proceed on the basis of any of these, the other(s) being deleted. If the applicant has not yet done so, the examining division should at the beginning of substantive examination, if it maintains the objection of lack of unity (see C‑III, 3.1.1), invite the applicant to state on which invention the prosecution of the application should be based and to limit the application accordingly by excising those parts belonging to the other inventions. For the latter inventions, the applicant may file divisional applications (see C‑III, 3.2)."

      Delete
  13. Hi everyone,
    Q8.4- According to Guidelines I undertand that always the consent of proprietor of the patent is required, so I consider the answer should be FALSE



    "Grounds for opposition submitted for the first time on appeal may be considered only with the patent proprietor's consent. If the patentee has not consented to a fresh ground's introduction into the proceedings, the board's decision must not deal with it in substance at all and may mention only that it has been raised (see G 10/91 and G 9/91, OJ 1993, 420; G 1/95, OJ 1996, 615).

    In T27/13 the board held that the objection under Art. 83 EPC in the statement of grounds constituted a fresh ground for opposition under Art. 100(b) EPC that was outside the legal framework of the opposition proceedings. In the light of G 10/91's observations on the legal character of opposition and opposition appeal proceedings and the particular importance it attached to grounds for opposition in that they established the legal framework within which substantive examination of the opposition was in principle to be conducted, the board did point out that it could find the present appeal admissible only if the same ground for opposition was raised when the appeal was filed. Only then could there be any discussion of the possibility of introducing a fresh, additional ground for opposition. Hence the appeal was inadmissible."

    ReplyDelete
    Replies
    1. See section III.P.2 of the Case Law Book that discusses G 1/94 and specifically discusses this exact scenario. None of the cases that you have cited relate to intervention of an assumed infringer, but instead discuss whether someone who is already party to the proceedings can introduce new grounds of opposition (i.e. the situation in questions 8.1 to 8.3)

      Delete
    2. G 1/94 is very clear on this: an intervener can rely on any fround, also on non-raised grounds. Further, as a rule, the opposition will be remitted back to first instance if the intervener introduces new grounds.

      G 1/94, headnote: "Intervention of the assumed infringer under Article 105 EPC is admissible during pending appeal proceedings and may be based on any ground for opposition under Article 100 EPC."

      G 1/94, reason 12-13:
      "12. One point, however, which was fully argued before the Enlarged Board in the present case and which needs to be clarified in this context because of its close connection with the principle issue of intervention in appeal proceedings, is the question, whether an assumed infringer may raise new grounds for opposition, which have not been considered in the previous proceedings before the Opposition Division. As appears from paragraph V of the Summary of Facts and Submissions, the intending interveners and the Patentee took contrary views on this matter.

      13. As explained by the Enlarged Board of Appeal in its opinion in case G 10/91 (OJ EPO 1993, 420), the purpose of the appeal procedure inter partes is mainly to give the losing party a possibility to challenge the decision of the Opposition Division on its merits, and it is not in conformity with this purpose to consider grounds for opposition on which such decision has not been based. The raising of new grounds for opposition by an assumed infringer, intervening in appeal proceedings, certainly does not fit with this basic concept of the appeal procedure. However, the purpose of intervention is to allow the assumed infringer to defend himself against the Patentee's action. Therefore, to prevent him from making use of all available means of attacking the patent, which he is accused of infringing, including the raising of new grounds for opposition under Article 100 EPC not relied upon by the proper Opponent, would run contrary to this purpose of intervention. Furthermore, this would involve the risk of conflicting decisions on the validity of European patents in the EPO and national courts, such decisions being based on different facts and grounds. Therefore, the Enlarged Board takes the view that intervention under Article 105 EPC in pending appeal proceedings may be based on any ground for opposition under Article 100 EPC. However, in application of what has been stated by the Enlarged Board in case G 10/91 for the exceptional situation of the introduction of new grounds in ordinary appeal proceedings, if a fresh ground for opposition is raised by the intervener, the case should be remitted to the first instance for further prosecution, unless special reasons present themselves for doing otherwise, for example when the Patentee himself does not wish the case to be remitted."

      Delete
  14. Q2.2 PCT-1 can validly claim priority from FR-1.

    There is no mention that FR-1 is a first filing. So, in my opinion, based on the information given in Q2, the answer can be "FALSE".

    Rong

    ReplyDelete
    Replies
    1. This is "CAN", it is possible, so TRUE...

      Delete
    2. @Rong, I think if you put such thinking to every question in the Pre-EQE, then you will find lots of questions cannot be answered in a certain way...

      Delete
    3. Thx Rong for your comment. Thx Monk for your response, I agree.
      I appreciate that candidates discuss on these blogs and not just consume the information! Discussion and sharing doubts is an efficient way to get to understanding and agreement (and sometimes agreeing to disagree or agreeing that there are unclarities).

      Rule 22(3) IPREE: Candidates shall accept the facts given in the examination paper and limit themselves to those facts. Whether and to what extent those facts are used shall be the responsibility of each candidate.

      In my opinion this means FR-1 shall be considered the first filing in this case, as you would have needed more facts to be able to conclude that it is not. if you think it may not be, you do not limit yourself to the facts given.

      And yes, it would have been nice if the question would have made it explicit. But if questions need to give all "not"-information, they explode and they get obscured. I and many candidates appreciate that questions are concise, with all the facts needed, with the knowledge that the answer cannot be affected by a fact that is not given (e.g., here, a speculation that there is an even earlier application). In case of doubts, I recommend candidates to read the question with the "willingness to understand" and with the knowledge that it is an exam question so that certain information plays a certain role - that often helps to solve alleged unclarities.

      Delete
    4. Thanks Roel for providing official guidelines for answering questions!

      Indeed, because for example, candiate may argue: what if there is an interruption? I have imaged an interruption, so every statement regarding time limit, due day is false. That would be too funny ;)

      Delete
    5. To prevent misunderstanding: only Rule 22(3) IPREE is official guidelines (even stronger than that, they are implementing regulationsm they are part of the legal framework of the exam), which all candidates already know ;)

      The last paragraph of my previous post is not official, they present my view and my recommendations.

      Delete
  15. Q6.2: An objection from the examining division under Article 83 EPC regarding embodiment X1 can be overcome by filing additional technical information.

    I struggled with answering either T or F. If the applicant wants to keep the filing date, then the answer clearly should be "F". However, if the applicant files the technical information as missing parts, and a new filing date will be accorded when those missing parts are filed, then the answer could be "T". Due to the ambiguity in the question I have (presumably) wrongly answered "T".

    ReplyDelete
    Replies
    1. @Thomas, if you refer back to the question itself, it already says "as essential technical information is missing". So to answer Q6.2, there is no missing parts that can be filed (No priority is mentioned for the EP application either, so you have to assume there is no priority).

      Maybe you are confused this situation with Q6.3 where it assumes that the essential info is in the abstract and it might lead you to think what if the essential info is in the priority... But from the background information and Q6.2 alone, I don't think you can or should think this way.


      BTW, I found the random orders in the question during the e-EQE is very confusing.I understand it is for couter-cheating, but sometimes such disorder of the questions may have unconformtable influence. Especially when in the claims analysis part, the question regarding whether Claims II.7 is novel over D1 is in the FIRST question. Then, I read the whole claims and compared one by one, and had an answer. Then, in the next question, I was asked to check Claims II.2! I feel so fooled, because I completely forget about Claims II.2, had to go back and compare again. That wasted a lot of time -> I know that I should have read through the whole four questions and come up with a better order to answer the question...

      Delete
    2. Hi Thomas,

      I see your reasoning, but any missing parts must be filed within 2 months of filing (or notification of missing parts). The question states that there has been an objection by the examining division. Examination does not normally start until at least 18 months after the priority. Realistically speaking there are no situations where you would get an objection from the examining division within 2 months of filing.

      Delete
    3. @James: thanks for another insight! I found your thorough thinking is more reasonable!

      Delete
    4. @James and Mink: you seem to read somethin in the statement that is not written:

      "If the examining division does not raise any objection under Article 83 EPC, lack of sufficiency of disclosure will not be a valid ground for opposition against the patent granted on the basis of EP-X."

      It simply says that if -at some time, in the future- ... It nowhere says that that objection would come in the first 2m...

      Delete
    5. ...and from the Instructions:

      1.a) "Each statement within a question is to be considered independently of the other statements."

      http://documents.epo.org/projects/babylon/eponot.nsf/0/F174C552F0E5DC4EC1258679004B17AF/$FILE/Notice_MarkingScheme_PreEx2021_EN.pdf

      Delete
    6. Hi Roel, I think you may have mixed up the numbering of the questions. Your comments apply to 6.4, whereas Thomas is asking about 6.2 :)

      Delete
    7. @Roel, thanks for pointing out the independency of each statement.
      Yes, I agree that in the legal part, random order of the statement should not be a problem.
      However, in the claims analyais, for example, the question regarding novelty in this kind of order:
      1. Cl. II.7
      2. Cl. II.2
      3. Cl. II.4
      4. Cl. II.6

      Such a disorder is at least unconfortable. Because it took me at least 5-10 minute to go though the whole claims set to answer, and I could have done the rest three statement at the same time.

      The same disorder happens for A.123(2) question again. But this time I learnt my lession, so I followed the claims sequence to do the questions.

      Delete
    8. The randomization is quite a pain for our discussions. That is why we have added the statements in full in our blog post, where they are presented in the sequence that one candidate actually got in the exam... By giving the statements, we hope that we are always discussing the same statement, but that may sometimes go wrong...

      Delete
    9. Correction: it took me 5-10 mintutes to have an answer for 1. Cl. II.7, and then I was completely upset that I see the next question is to check Cl. II.2 ...

      Delete
    10. @James 3 March 2021 at 17:12

      O, sorry... I agree with your comment of 3 March 2021 at 16:52: will be outside the 2m from filing.

      6.2 "An objection from the examining division under Article 83 EPC regarding embodiment X1 can be overcome by filing additional technical information."

      Delete
  16. What about 4.4 (The period for entering the European phase will expire on 02 October 2023)?

    ReplyDelete
    Replies
    1. Please clarify your question. What is your worry/issue/question? Does it relate to why 2/10/32 could be a possible date to consider?

      If you would have started the 31m period at the filing date rather than the priority date, you would have gotten from 1/3/2021 +31 m --> 1/10/23. There was no 2023 calendar, so you could not know that this was a Sunday, but if you did know or guess that, you would have concluded that the 31m extends to 2/10/23 under R.134(1).
      But as the 2023 calendar was lacking, you would probably have been lucky: you would have conclude expiry on the 1/10/23, and concludes FALSE... but for the wrong reason.

      Delete
  17. What about 7.4 (Professor N has the right to make oral submissions during the oral proceedings, since she must be considered by the examining division to represent the “person skilled in the art”.)?

    He can make submissions as an expert, isn't it? Or as a witness?
    I answered TRUE.

    ReplyDelete
    Replies
    1. My first impression about this statement is that the since part is so wrong. Even a Nobel prize winner has to follow EPC before the EPO. Then the first part is also apparently wrong. Accompany person should be requested in advance and whether allowed to speak is at the discretion of the exam board. I don't see any valid point that can overrule the judgement about this statement.

      If you are referring to expert or witness, then there is GL E-IV, 1.6.2
      Witnesses and experts not summoned.

      Delete
  18. What about 10.1 (In order to receive a refund of an additional search fee paid in the European phase, the applicant must pay the additional search fee under protest)?

    I cannot find a definition of protest in the EPC.

    Why would the definition of that term in the PCT define the meaning in the EPC regional phase? In PCT only for international search.

    A-X, 10.2.2 says: 'If an applicant, following a communication from the search division, has paid a further search fee but the examining division, at the applicant's REQUEST, has found that there was no justification for charging the further search fee, the latter will be repaid.'

    Can this REQUEST not be considered a protest?

    ReplyDelete
    Replies
    1. It is false exactly because there is no such thing like protest in EPC. In this question, the word "protest" in Q10.1 is exactly the same as the protest in the background, so if you have the knowledge clearly in mind, you immediately notice this cannot be right. The request to refund additional search fee in EPC and the protest under PCT are two different things.

      However, if you insist to consider protest as the request, because they share the same nature, I have no better argument. This kind of interpretation is just an argument trying to get your point on this question, which is not at my hand ;)

      Delete
    2. The question asks whether the fee must be paid under protest, not whether you can protest or not. Since the refund is decided at a later date by the Examining Division, a protest is not required when paying the fee

      Delete
    3. @Anonymous 3 March 2021 at 19:57:
      Even if you would consider the request from A-X 10.2.2 a "protest", the answer would be FALSE as there is no need to make that request at the time of payment of the further search fee(s).

      But in the context of this question, where EPC and PCT aspects of non-unity are considered, it seems far-fetched and incorrect to give your own meaning of a term that is well-defined in one of the systems (PCT) but has no equivalent in the other (EPC).

      Delete
  19. What about 2.4 (Anna can validly file the priority declaration in May 2021)?

    I answered FALSE as the time limit to file the priority declaration expires on 6 June 2021, and not in May.

    ReplyDelete
    Replies
    1. Yes, the time limit for filing the declaration expires in June. Therefore she can file it in May, since she can file it anytime between the filing date and the due date.

      Note that the questions does NOT ask if the time limit expires in May. It just asks if filing is possible in May, which is TRUE, since the time limit expires later that that.

      Delete
    2. Similar to someone mentioned above for interpreting the word of CAN.

      I think people should distinguish CAN, MUST, SHALL, MAY as a representative candidate

      Delete
  20. I think the answer of Q1.3 would be True. Art. 86 (2) The obligation to pay renewal fees shall terminate with the payment of the renewal fee due in respect of the year in which the mention of the grant of the European patent is published in the European Patent Bulletin. Art. 141 (1) Renewal fees for a European patent may only be imposed for the years which follow that referred to in Article 86, paragraph 2. Therefore, the renewal fee of the fifth year should be paid to the EPO

    ReplyDelete
    Replies
    1. I don't know what to say because you find the correct legal basis. Maybe you get the wrong fifth year....

      Delete
    2. Hello Unknown - remember that the patent year runs *from* the anniversary of filing, not *to* the anniversary of filing. Thus, the year in which the mention of the grant of the European patent is published in the European Patent Bulletin for this question is the 4th year. Hope this helps!

      Delete
    3. made the same mistake as unknown. need to practice renewal fees!

      Delete
  21. We have compiled a version of the full paper (in English); it is available via the link that I added to the blog post.

    In our version, the sequence of the questions and statements corresponds to the order used in this blog, as some of the candidates got them in the real exam; other candidates obtained the questions (in the legal parts) and the statements within a question (in the legal parts as well as in the claims analysis parts) in different order.

    ReplyDelete
  22. Reminder:

    Please do not post your comments anonymously - it is allowed, but it makes responding more difficult and rather clumsy ("Dear Mr/Mrs/Ms Anonymous of 05-03-2021 16:56"), whereas using your real name or a pseudonym is more personal, more interesting and makes a more attractive conversation.

    You do not need to log in or make an account - it is OK to just put your (nick) name at the end of your post.

    Thanks!

    ReplyDelete
  23. I could not answer
    "The period for entering the European phase will expire on 02 October 2023."

    To conclude whether the 31m limit expired on 1 October 2023 or 2 October 2023, I had to check whether 1 October 2023 is a day on which the EPO is closed, so that it would indeed be 2 October 2023. But the paper had no calendar of 2023.

    ReplyDelete
    Replies
    1. You have used the filing date for your calculations instead of the priority date. The 31m limit expires in 2022, which is all you need to know to answer the question. (See Roel's comment at 16:10 on 4 March)

      Delete
  24. When do we get our marks?

    When do we get the Examiner’s Report? Same day, earlier or later?

    Which statements will accept both True and False?

    ReplyDelete
    Replies
    1. In 2018 and 2019, Pre-Exam results became available 3-4 weeks after the exam:
      - 2019: Pre-Exam on 25/2/2019 - results on 12/3/2019; Examiner's report on same day;
      - 2018: Pre-Exam on 26/2/2018 - results on 21/3/2018; Examiner's report on same or next day (and updated in June due to -presumably interlocutory revision of- appeals).

      A bit of a delay may be in your advantage: then the Committee and Examination Board have more time to take any input into account to consider, for example, neutralization of some statements.
      So although the marking can in principle be quick now since the answers are electronically (in earlier exams, the answer sheet with the colored circles had to be collected, scanned, checked - partially by hand, ...), the careful marking and evaluation process needs some time.

      Our post and the comments give some indications as to which statements would be candidates for neutralization. In the legal part, I think 6.3, 10.2 and 10.4 (in our version) are candidates for neutralization - I refer to above for arguments for TRUE as well as for FALSE for those statements.

      Delete
  25. Where can I find a copy of the Pre-Examination paper? The EQE Compendium only had the 2019 and earlier papers.

    ReplyDelete
    Replies
    1. The Pre-Exam 2021 paper is not (yet) available on the Compendium pages.

      We compiled a version of the Pre-Exam 2021 paper, in English, with the questions and statements in a sequence that a real candidate received:
      you can get our version via the link in the blog post itself (also in the first impressions as well as in the claims analysis blog post).

      Enjoy! And please feel invited to post any comments about the paper in tour blogs.

      Delete
  26. For those in the know, how do we know when the results are released? Are we notified by email? Or, are the results just published here: https://www.epo.org/learning/eqe/statistics.html and the news spreads?

    ReplyDelete
    Replies
    1. Last years, the list was published in the EQE website and shortly thereafter all candidates were individually informed by a letter (by post).

      Now, you will presumably get your pass/fail decision via myEQE. See OJ 2020, A140 (https://www.epo.org/law-practice/legal-texts/official-journal/2020/12/a140.html)

      Delete
  27. I just got my result on myEQE, two points below the estimated point based on delta's answer in worst case. It seems that no neutralization was done...Now waiting for the official answer to be published...

    ReplyDelete
  28. Some candidates informed me that the individual Pre-Exam results are available via MyEQE now).

    The Examiner's Report is not yet available, nor is a full list of marks which would allow to determine the pass rate.

    ReplyDelete
  29. The Examiner's report is now available.

    In the legal part, Q.10 (in our numbering and in the numbering in the Examiner's report) was completely neutralized. The Examiner's report provides:

    "Question 10 mistakenly refers to applicant B and applicant A while it was intended to refer to only one applicant B different from applicant A of question 4. To avoid any disadvantage due to this mistake, it has been decided that for this question 10 all answers are awarded the full marks of 5 points."

    No other statements were neutralized in the legal part.

    Our answers to Q.1-Q.9 correspond to that of the Examiner's report.

    Note that in the Examiner's Report, Q.7 and 8 are swapped compared to our version.
    Also, the statements are generally in a different order.

    Note that ion Q.1, our version had the somewhat strange order "4th yr - 3rd yr - 5th yr - div" whereas the Examiner's report had the natural order "3rd yr - 4th yr - 5th yr - div" - which probably has less risk of going wrong. Some candidates m,ay have had an even more awkward order, e.g., "5th - 3rd - div - 4th" which seems significantly more easily prone to errors.

    ReplyDelete
    Replies
    1. I am very confused by 6.3 (of the Examiner's Report). There does not appear, to me, to be anything in Q6.3 that prohibits re-dating as a consequence of filing missing parts (and thus not contravening A123(2)).

      Delete
    2. Exactly. The whole question specifically uses the phrase "missing parts" and this is not addressed at all by the Examiners.

      Now that the results are out, I will be a little more open with my thoughts on that question. My feeling is that missing data as such would not be considered as a missing part. J 27/10 is the only piece of case law on the subject and whilst it doesn't provide a clear cut answer, my feeling is that the incorporation of missing data/sentences that are not for examples a complete missing page, would not be accepted. (I didn't mention this before, as it slightly undermined our other arguments on the subject).

      What annnoys me slightly is that they haven't even given an indication of what the answers for question 10 would be had there not been a typo regarding Applicant A /Applicant B. Given our lengthy discussion above, it is rather frustrating not to know what the correct answer was supposed to be.

      Delete
    3. 6.3 (also 6.3 in our blog version) has been extensively discussed in our blog post and the blog comments above.

      Delete
    4. Hi Roel, from my side at least, I am not trying to reopen the discussion on this question, but we were simply pointing out that the argumentation as presented by the Examiner appears to be completely unrelated to the question that was asked.

      The question asked was whether the information could be included "as missing parts". The Examiner procedes to discuss Article 85 EPC and Article 123(2) EPC, i.e. whether an amendment can be supported by the appication as filed. This is a completely different question as to whether the information can be included "as missing parts".

      There are a number of points in the Examiner's report where (irrespective of whether I agree with the answer or not), the argumentation provided is clearly wrong (or not related to the question posed in the exam). See Nico's comments on GL 4-IV 4.6.1 for example.

      I understand that there are always issues in the pre-EQE regarding interpretation of the question (no exam can be perfect), but I find it a little worrying that the Examiner's argumentation is of such poor quality (and sometimes incorrect)

      Delete
    5. Hi James,

      I agree that it is strange that the argumentation does not refer to missing parts at all, but only to Art.123(2), i.e. to amendment:

      1) Rule 56 should have been discussed also in my view as the statement is " can be incorporated into EP-X as missing parts", even while indicating a time limit ("within one month from the date of filing");

      2) at this stage, amendments are not possible as the search report is not yet drawn up and this not yet received by the applicant - Rule 137(1).

      So, the connection between the argument and the (second) legal principle in the claim (R.56; the first being Art.85) is missing. In DI, the argumentation given by the Examiner's Report would not score many marks.

      For reference: the statement (6.3) and argument from the Examiner's Report:

      "If the missing essential technical information regarding embodiment X1 is present in the
      abstract, the essential technical information can be incorporated into EP-X as missing
      parts within one month from the date of filing.

      FALSE - The abstract is not part of the disclosure (Article 85 EPC) so including the
      essential technical information from the abstract would offend Article 123(2) EPC."

      Delete
    6. For additional reference, the wording of R.56(1)&(2):

      Rule 56 Missing parts of the description or missing drawings
      (1) If the examination under Article 90, paragraph 1, reveals that parts of the description, or drawings referred to in the description or in the claims, appear to be missing, the European Patent Office shall invite the applicant to file the missing parts within two months. The applicant may not invoke the omission of such a communication.
      (2) If missing parts of the description or missing drawings are filed later than the date of filing, but within two months of the date of filing or, if a communication is issued under paragraph 1, within two months of that communication, the application shall be re-dated to the date on which the missing parts of the description or missing drawings were filed. The European Patent Office shall inform the applicant accordingly.

      Delete
    7. Roel, exactly, agreed. It worries me that future candidates will use the Examiner's Report to guide their learning, and this is a clear error that will no doubt mislead a candidate's understanding. Regarding your point below 2-Apr 9:55, I was simply noting to James that there is nothing excluding that a whole page is missing. I was not taking a view on what does or does not constitute a missing part.

      Delete
  30. James, I don't quite agree with your middle paragraph because there is nothing to suggest that there is missing data 'as such'. I haven't read the case or considered the substance of your argument. However, all we are told is that there is missing 'essential technical information' - that could well be because a whole page was missing and it just happened that the essential technical information was on that missing page.

    I hope that there is no one with a mark of 68/69 who answered True for Q6.3.

    ReplyDelete
    Replies
    1. Fair point. How much you should read into potential situations that are not discussed in the question is debatable, but I think we can all agree that this question was HORRIBLY written. It doesn't matter for me in terms of pass/fail, but I think it should definitely have been neutralized.

      Delete
    2. Hi Adan, I do not agree fully with your comment, or I do not understand it.

      You seem to overlook that the question says:
      "Embodiment X1 is not sufficiently disclosed as essential technical information is missing"

      Further, "missing parts of the description" acc R.56 does not need to be a complete page, it can also be some smaller part, e.g., just a part of a sentence.

      Delete